Inscription / Connexion Nouveau Sujet
Niveau Licence Maths 1e ann
Partager :

point fixe et contraction

Posté par
tazia
04-07-09 à 13:36

Bonjour!

J'espère que vous pouvez m'aider!

Soit (X,d) un espace vectoriel normé. Soit :XX une contraction lorsque x,yX: xy et
d((x),(y))< d(x,y)

a) il faut que je trouve une contraction dans avec d(x,y)=|x-y|, qui n'a pas de point fixe. il faut constater (x)-x (et quelles sont les caractéristiques qu'elle doit avoir?)

MERCI d'aance!

Posté par
dolma
re : point fixe et contraction 04-07-09 à 15:26

Salut Tazia,

Bon, c'est la première fois que j'entends parler de contractions donc je ne peux pas dire que ce que j'ai trouvé est a 100% sur, mais ça m'a l'air bon.

Déjà, tu cherches une application : . Tu peux donc tracer la courbe correspondante y=(x).

Maintenant traces la droite y=x et tu te rends vite compte que toute courbe de "pente" (en valeur absolue) inférieure à 1 est une contraction et toute courbe de "pente" (toujours en valeur absolue) supérieure à 1 n'en est pas une.
En effet, comme ici la distance d est définie par 3$d(x,y)=\|x-y\| il te suffit d'ecrire la valeur absolue du taux de variation de ta courbe qui est :

4$\|\frac{\Phi (x)-\Phi (y)}{x-y}\|\ =\ \frac{\|\Phi (x)-\Phi (y)\|}{\|x-y\|}\ =\ \frac{d(\Phi (x)-\Phi (y))}{d(x-y)}

On voit bien que si c'est inferieur à 1, c'est bien la définition de la contraction que t'as donnée au debut .

Maintenant pour qu'il n'y ait pas de point fixe, il suffit que ta courbe ne coupe pas y=x.

Et voila, donc, toute courbe qui admet en tout point un taux de variation tel que || < 1 et ne coupant pas la droite x=y represente une contraction de n'ayant pas de point fixe.

Le coup de constater (x)-x , je suppose que c'est pour dire que comme la courbe ne doit pas couper y=x, il faut donc que (x)-x soit de signe constant (et donc ne s'annule pas).

Voila voila, j'espère que ca t'aidera. (et surtout que c'est pas faux )


PS : J'ai mis \Phi au lieu de dans les équations mais c'est la même chose, j'ai juste pas trouvé le sous LaTeX

Posté par
dolma
re : point fixe et contraction 04-07-09 à 15:28

Ooops, dans la formule c'est 4$\frac{d(\Phi (x),\Phi (y))}{d(x,y)} et pas 4$\frac{d(\Phi (x)-\Phi (y))}{d(x-y)}.

Posté par
caypak
re : point fixe et contraction 04-07-09 à 17:33

Tazia,
remarquons tout d'abord que pour d(x,y)=/x-y/ ,d définit une distance sur R.Alors (R,d) est un espace métrique et de surcroit complet.Par suite,toute appliction contractante de R à valeurs dans R admet un unique point fixe d'après bien entendu le théorème des points fixes de Banach.

En revenant donc à l'exercice,on ne trouvera jamais une contraction de R qui n'a pas de point fixe.Et déjà il faut rectifier la def donnée:l'hypothèse
d(f(x),f(y))<d(x,y) n'implique pas que f est une contraction:f est une contraction sur X si pr tt (x,y) dans XxX ,d(f(x),f(y))<=kd(x,y) avec 0<=k<1. merci et bye.                                              

Posté par
tazia
re : point fixe et contraction 04-07-09 à 21:07

je vous remercie énormément de votre aide...(en fait il s'agit d'une contraction point par point)n'ayant pas de point fixe c'est pas évident À montrer..

Posté par
tazia
re : point fixe et contraction 04-07-09 à 21:10

En cours on a parlé de Banach (mais bon le cours est en allemand)

Posté par
elhor_abdelali Correcteur
re : point fixe et contraction 04-07-09 à 22:08

5$\fbox{\varphi\;:\;\mathbb{R}\to\mathbb{R}\\\;\;\;\;x\to\frac{x+\sqrt{x^2+1}}{2}} sauf erreur bien entendu

Posté par
tazia
re : point fixe et contraction 04-07-09 à 23:46

je ne vois pas trop pourquoi ca n'admettrait pas de point fixe?!

Posté par
elhor_abdelali Correcteur
re : point fixe et contraction 05-07-09 à 00:42

Eh bien tu résouds l'équation 4$f(x) = x pour t'en convaincre

Posté par
Camélia Correcteur
re : point fixe et contraction 05-07-09 à 14:12

Bonjour

Voici aussi ma contribution. (pas la même que celle d'elhor.

Il y a deux idées: d'abord pour être sur qu'une fonction est contractante, on peut la prendre dérivable avec dérivée strictement inférieure à 1 (en valeur absolue)
et faire marcher le théorème des accroissements finis.

Ensuite, une fonction f est sans point fixe si et seulement si elle est de la forme f(x)=x+g(x) où g ne s'annule pas.

Avec ça, voici un exemple (après un peu de bricolage)

\varphi(x)=x-\frac{1}{2}\arctan(x)+1

Evidemment pas de point fixe, et

\frac{1}{2}\leq 1-\frac{1}{2(1+x^2)}< 1

Posté par
tazia
re : point fixe et contraction 06-07-09 à 20:25

Je vous remercie énormément Camélia et Elhor! grâce à vous j'ai compris!!!(mon message est un peu tard )

Posté par
elhor_abdelali Correcteur
re : point fixe et contraction 07-07-09 à 23:01

De rien tazia

la fonction x\to\varphi(x)-x étant continue et ne s'annulant pas sur \mathbb{R} elle y garde un signe constant et on a donc :

\fbox{*} soit \varphi(x)-x>0 pour tout réel x comme c'est le cas par exemple pour \;\varphi\;:\;x\to\frac{x+\sqrt{x^2+1}}{2}

noter que dans ce cas \varphi est strictement positive au moins sur \mathbb{R}+ et \lim_{+\infty}\;\varphi=+\infty

\fbox{*} soit \varphi(x)-x<0 pour tout réel x comme c'est le cas par exemple pour \;\varphi\;:\;x\to\frac{x-\sqrt{x^2+1}}{2}

noter que dans ce cas \varphi est strictement négative au moins sur \mathbb{R}- et \lim_{-\infty}\;\varphi=-\infty sauf erreur bien entendu



Vous devez être membre accéder à ce service...

Pas encore inscrit ?

1 compte par personne, multi-compte interdit !

Ou identifiez-vous :


Rester sur la page

Inscription gratuite

Fiches en rapport

parmi 1675 fiches de maths

Désolé, votre version d'Internet Explorer est plus que périmée ! Merci de le mettre à jour ou de télécharger Firefox ou Google Chrome pour utiliser le site. Votre ordinateur vous remerciera !